June, Gavyn and Alex share some sweets in the ratio 5:5:3. June gets 22 more sweets than Alex. How many sweets are there altogether?

Answers

Answer 1

Answer:

Let June's sweets be 5x.

Then Gavyn's sweets will be 5x.

And Alex's sweets will be 3x.

5x = 3x + 22

2x = 22

x = 11

So June has 5 x 11 = 55 sweets

Gavyn has 5 x 11 = 55 sweets

And Alex have 3 x 11 = 33 sweets

Total

= 55 + 55 + 33

= 143 sweets


Related Questions

Approximately 10% of all people are left-handed. Consider a grouping of fifteen people.

Write the probability distribution

Answers

Answer:

Step-by-step explanation:

Answer:

15*.1=1.5

so either one or two people of the 15 would be left handed

write the equation of the line shown in the graph above in slope-intercept form

Answers

y = mx + c
y= -2/3x + 1
1= c
Answer: y= -2/3x+1


Step 1: Write slope-intercept form equation

To start answering this question, it helps to write out the example equation for slope-intercept form. Let’s do this now.

y=mx+b

Keep in mine that the y represents the answer, the m represents the slope, the x represents the variable, and the b represents the y-intercept.

Step 2: Determine the y-intercept

I find that it is easiest to determine the y-intercept first. This is the b in the equation. The y-intercept can be find by looking at the y-axis, and seeing where the line intersects with the axis. In this case, it seems to be at 1. Let’s substitute this into our equation now.

y=mx+b
y=mx+1

Step 3: Determine slope

For the final thing that we need to fill in, we need to find the slope. The slope is the m in the equation, and can be found by the formula of rise/run. Choose any two points, and see how far they are from each other up and down (rise), as well as left and right (run).
For the sake of this problem, we will work with (0,1) and (-3,3). There is a difference in the rise of 2 and in the run of 3. This makes the rise/ run be 2/3. Let’s substitute this now.

y=mx+1
y= 2/3x+1

The slope will also need to be negative since the line on the graph descends. Let’s change this to a negative now.

y= -2/3x+1


This is your answer. Hope this helps! Comment below for more questions.

help pls, stuck on this

Answers

Answer:

Step-by-step explanation:

When P = 2l + 2w is solved for w, the result is:?

Answers

Answer:

[tex]\frac{p-2l}{2}[/tex]

Step-by-step explanation:

move the 2l to the other side by subtracting 2l on both sides. you get P - 2l = 2w. now divide both sides by 2 to get the answer.

a bag contain 3 black balls and 2 white balls.
1. A ball is taken from the black and then replaced, a second is taken. what is the probabilities that.

(a) there are both black,
(b)one is black one is white,
(c) at lease one is black,
(d) at most one is one is black.

2. find out if all the balls are chosen without replacement.

please kindly solve with explanation. thank you.​

Answers

Answer:

Step-by-step explanation:

Total number of balls = 3 + 2 = 5

1)

a)

[tex]Probability \ of \ taking \ 2 \ black \ ball \ with \ replacement\\\\ = \frac{3C_1}{5C_1} \times \frac{3C_1}{5C_1} =\frac{3}{5} \times \frac{3}{5} = \frac{9}{25}\\\\[/tex]

b)

[tex]Probability \ of \ one \ black \ and \ one\ white \ with \ replacement \\\\= \frac{3C_1}{5C_1} \times \frac{2C_1}{5C_1} = \frac{3}{5} \times \frac{2}{5} = \frac{6}{25}[/tex]

c)

Probability of at least one black( means BB or BW or WB)

 [tex]=\frac{3}{5} \times \frac{3}{5} + \frac{3}{5} \times \frac{2}{5} + \frac{2}{5} \times \frac{3}{5} \\\\= \frac{9}{25} + \frac{6}{25} + \frac{6}{25}\\\\= \frac{21}{25}[/tex]

d)

Probability of at most one black ( means WW or WB or BW)

[tex]=\frac{2}{5} \times \frac{2}{5} + \frac{3}{5} \times \frac{2}{5} \times \frac{2}{5} + \frac{3}{5}\\\\= \frac{4}{25} + \frac{6}{25} + \frac{6}{25}\\\\=\frac{16}{25}[/tex]

2)

a) Probability both black without replacement

  [tex]=\frac{3}{5} \times \frac{2}{4}\\\\=\frac{6}{20}\\\\=\frac{3}{10}[/tex]

b) Probability  of one black and one white

 [tex]=\frac{3}{5} \times \frac{2}{4}\\\\=\frac{6}{20}\\\\=\frac{3}{10}[/tex]

c) Probability of at least one black ( BB or BW or WB)

 [tex]=\frac{3}{5} \times \frac{2}{4} + \frac{3}{5} \times \frac{2}{4} + \frac{2}{5} \times \frac{3}{4}\\\\=\frac{6}{20} + \frac{6}{20} + \frac{6}{20} \\\\=\frac{18}{20} \\\\=\frac{9}{10}[/tex]

d) Probability of at most one black ( BW or WW or WB)

 [tex]=\frac{3}{5} \times \frac{2}{4} + \frac{2}{5} \times \frac{1}{4} + \frac{2}{5} \times \frac{3}{4}\\\\=\frac{6}{20} + \frac{2}{20} + \frac{6}{20} \\\\=\frac{14}{20}\\\\=\frac{7}{10}[/tex]

X⁴-6x²-7-8x-x² what is the answers

Answers

Answer:

X⁴-7x²-8x-7

Step-by-step explanation:

Jimmy thought he had purchased 7 folders, but purchased 6. What was his percent error?

Answers

Answer:

Step-by-step explanation:

Percent Error = | Actual Yield-Theoretical/ Theoretical Yield | *100%

Error= |-1/7|*100%= 14.29%

rotation 180 degrees about the origin.​

Answers

Answer:

Take the picture you uploaded.

Click the rotate button twice.

Done

Which one is a better deal?
paying $2.88 for a 12 roll package of toilet paper
paying $1.20 for a 6 roll package of toilet paper

Answers

Answer:

paying $1.20 for a 6 roll package of toilet paper

Step-by-step explanation:

to find the answer, double 6 to equal 12 and double the price as well. therefore, it is 2.40. since 2.40 is cheaper than 2.88, it is a better deal.

Answer:

paying $1.20 for a 6 roll package

Step-by-step explanation:

$2.88 divided by 12 = $0.24
$1.20 divided by 6 = $0.20

Hence, i think the best deal is paying $1.20 for a 6 roll package

Is this right?? If not please tell me the answers

Answers

yes all of these are correct

Answer:

TAMA PO

Step-by-step explanation:

Paki brainliest nadin po

Happy buys 5 litres of milk on Monday and uses a fifths of it. On Tuesday she uses
half of what is left. How many millilitres are left after Tuesday?​

Answers

it is 2000 ml after she drank it on Tuesday

Answer:

2000 mL

Step-by-step explanation:

1. The amount of milk left after 1/5 is drank is 4 litres, which is 4000 mL.

2. She drinks half of that, so divide 4000 mL by two.

2000 mL are left after tuesday

Use the relationship among the three angles of any triangle to solve. Two angles of a triangle have the same
measure and the third angle is 27° greater than the measure of the other two. Find the measure of each angle.

Please help :)

Answers

Answer:

51°,51°,78°

Step-by-step explanation:

The sum of angles in a triangle add up to 180°

The mean of a data set is observed to be very different from its median, representing a strong skewness. However, the 1.5 IQR rule reveals that there are no outliers. Which of the following is correct, if the sample size is 100?

a. A normal quantile plot of the data follows a diagonal line, and the t-procedure is appropriate to use.
b. A normal quantile plot of the data does not follow a diagonal line, and the t- procedure is not appropriate to use.
c. A normal quantile plot of the data follows a diagonal line, and the t-procedure is not appropriate to use.
d. A normal quantile plot of the data does not follow a diagonal line, and the t- procedure is appropriate to use.

Answers

Answer:

a. A normal quantile plot of the data follows a diagonal line, and the t-procedure is appropriate to use.

Step-by-step explanation:

Central Limit Theorem

The Central Limit Theorem establishes that, for a normally distributed random variable X, with mean [tex]\mu[/tex] and standard deviation [tex]\sigma[/tex], the sampling distribution of the sample means with size n can be approximated to a normal distribution with mean [tex]\mu[/tex] and standard deviation [tex]s = \frac{\sigma}{\sqrt{n}}[/tex].

For a skewed variable, the Central Limit Theorem can also be applied, as long as n is at least 30.

For a proportion p in a sample of size n, the sampling distribution of the sample proportion will be approximately normal with mean [tex]\mu = p[/tex] and standard deviation [tex]s = \sqrt{\frac{p(1-p)}{n}}[/tex]

In this question:

Sample size of 100 > 30, which means that we use the Central Limit Theorem, and thus, the sampling distribution is approximately normal, following a diagonal line, and since the standard deviation of the population is not know, we use the t-procedure. Thus, the correct answer is given by option a.

How do I solve this?

Answers

Answer: 3

Step-by-step explanation: m=2 so 7.5x2 = 15

15/5 is 15 divided by 5 so the answer is 3

Answer: 3


Step 1: Rewrite equation

To start off, I find it helpful to rewrite the equation. This will give a better understanding of the problem we are working with.

7.5m/5

Step 2: Substitute information

Now that we know our equation, we can substitute the information we are given. In this case, we are made aware that m is equal to 2. Let’s change 7.5m to 7.5(2).

7.5m/5
7.5(2)/5

Step 3: Solve

To solve our problem, we must first start with the top part of the equation, which tells us to multiply 7.5 by 2. Then, we must divide that by the number on the bottom part of the equation, which is 5. Let’s do this now.

7.5(2)/5
15/5
3

This is your answer. The equation equals 3. Hope this helps! Comment below for more questions.

Evaluate 19C1 PLEASE HELP

Answers

Answer:

[tex]{19}C_1=19[/tex]

Step-by-step explanation:

We need to find the value of [tex]{19}C_1[/tex].

C stands for combination.

The formula of combination is as follows :

[tex]nC_r=\dfrac{n!}{r!(n-r)!}[/tex]

Here,

n = 19 and r = 1

So,

[tex]nC_r=\dfrac{19!}{1!(19-1)!}\\\\nC_r=\dfrac{19!}{1!\times 18!}\\\\nC_r=\dfrac{19\times 18!}{1!\times 18!}\\\\nC_r=19[/tex]

So, the value of [tex]{19}C_1[/tex] is 19.

Answer:

Your answer will be 19C1 =19

ANSWER ASAP IM BEING TIMED
IF I GET AN A ON THIS I WILL DO ANOTHER POINT FREE DROP, PLEASE SHOW YOUR WORK
The lengths of three sides of a quadrilateral are shown below:

Side 1: 1y2 + 3y − 6

Side 2: 4y − 7 + 2y2

Side 3: 3y2 − 8 + 5y

The perimeter of the quadrilateral is 8y3 − 2y2 + 4y − 26.

Part A: What is the total length of sides 1, 2, and 3 of the quadrilateral? (4 points)

Part B: What is the length of the fourth side of the quadrilateral? (4 points)

Part C: Do the answers for Part A and Part B show that the polynomials are closed under addition and subtraction? Justify your answer. (2 points)

Answers

Answer:

Part A

(1y^2+3y-6)+(4y-7+2y^2)+(3y^2-8+5y)

6y^2+12y-21

in the past year bill watch 64 movies that he thought were very good he watched 80 movies over the whole year of the movies he watched what percentage did he rate as very good​

Answers

Answer:

he rate it 16%

Step-by-step explanation:

64-80\100=16

Golf Scores In a professional golf tournament the players participate in four rounds of golf and the player with the lowest score after all four rounds is the champion. How well does a player's performance in the first round of the tournament predict the final score

Answers

Answer:

Mean scores.

Step-by-step explanation:

The golf player will score in the first round, according to these scores the golf player scores can be predicted. The golf player can perform high in first round but he may score lesser in the second round due to stress or mental pressure. The scores can be predicted taking mean of the scores and adding standard deviation to it.

U have to work out the value of a by the way

Answers

Answer:

Step-by-step explanation:

180-90=2b+b

90=3b

90/3=b

30=b

2b=2*30

   =60

180-90=a+a

90=2a

a=90/2

a=45

the answer is 45 degrees

hope it helps!!let me know if it does

Answer:

a= 15°

Step-by-step explanation:

> use the fact that the sum of angles in a triangle is 180°

> based on the picture in the small right triangle we have b° +2b° +90° =180°

b +2b +90 =180° , combine like terms

3b +90 = 180, subtract 90 from both sides of the equation

3b = 90, divide by 3 both sides of the equation

b = 30°

> angle b has a ray that continues as a line so it makes an 180° angle and we have the acute triangle so we can write that

a + a+ (180-b) =180, substitute b

2a + 180-30 =180, subtract 180 from both sides, and add 30 to both sides

2a=30, divide by 2 both sides

a= 15°

In July 2014 one Mexican peso was worth 0.075 U.S. dollars. How many Mexican pesos was $133.00 U.S. dollars worth?

Answers

Answer:

1,773.33 Mexican pesos

Step-by-step explanation:

Create a proportion where x was how many Mexican pesos it was worth:

[tex]\frac{1}{0.075}[/tex] = [tex]\frac{x}{133}[/tex]

Cross multiply and solve for x:

133 = 0.075x

1773.33 = x

So, it was worth approximately 1,773.33 Mexican pesos

if i need 90 square feet of tile and each piece of tile covers 0.34 square feet, how much do i need in pieces

Answers

Answer:

265

Step-by-step explanation:

9514 1404 393

Answer:

  265

Step-by-step explanation:

Let t represent the number of tiles needed. Then the area covered by those t tiles will be ...

  area = t·0.34 ft²

We want that area to be 90 ft², so we can solve this equation for t:

  90 ft² = t·(0.34 ft²)

  90 ft²/(0.34 ft²) = t ≈ 264.71

About 265 tiles are needed to cover 90 ft².

Find the area of the shaded region. Round to the nearest tenth. 11.1m 130°
Area = [ ? ] m²​

Answers

The area of the shaded region is 294.5 m².

What is the area of the entire circle?

The area of the entire circle is calculated as follows;

A = πr²

where;

r is the radius of the circle

A = π ( 11.1² )

A = 387.1 m²

The area of the sector is calculated as follows;

A = ( θ/360 ) πr²

A = ( 130/360 ) x π ( 11.1² )

A = 139.8 m²

The area of the triangle is calculated as follows;

A = ¹/₂ ( sinθ )r²

A = ¹/₂ ( sin 130 ) (11.1²)

A = 47.2 m²

Area of the unshaded region is calculated as;

A' = 139.8 m²  -  47.2 m²

A' = 92.6 m²

The area of the shaded region is calculated as follow;

A'' = 387.1 m² - 92.6 m²

A'' = 294.5 m²

Learn more about area of shaded region here: https://brainly.com/question/14989383

#SPJ1

Hello!
i need help with question 67 & 68

Answers

Answer:

67. A

68. D

Step-by-step explanation:

I don't remember exactly the explanation, but I recommend you try to learn more about number lines sometime when you aren't under stress from schoolwork, because they're pretty simple questions to answer once you get a better understanding of them!



Let f(x) = 2x + 8, g(x) = x² + 2x – 8, and h(x)
Perform the indicated operation. (Simplify as far as possible.)
(g - f)(2) =

Answers

the answer is (g-f)(2)

09:30 am - 4:30 pm minus 30 minutes?

Answers

Answer:

4:30

because 9:30 minus 4:30 = 5:00 and 5:00 minus 30 =4:30

which pair of fractions are equivalent? 2/3 and 12/9 20/40 and 45/ 55 20/40 and 4/8 5/5 and 25/50​

Answers

Answer:

[tex]\frac{20}{40} \ and \ \frac{4}{8} \ is \ equivalent[/tex]

Step-by-step explanation:

1.

[tex]\frac{2}{3} \ and \ \frac{12}{9} \\\\\frac{2}{3} \ and \ \frac{4}{3}\\\\Not \ equivalent[/tex]

2.

[tex]\frac{20}{40} \ and \ \frac{45}{55}\\\\\frac{1}{2} \ and \ \frac{9}{11}\\\\Not\ equivalent[/tex]

3.

[tex]\frac{20}{40} \ and \ \frac{4}{8}\\\\\frac{1}{2} \ and \ \frac{1}{2} \\\\Equivalent[/tex]

4.

[tex]\frac{5}{5} \ and \ \frac{25}{50} \\\\\frac{1}{1} \ and \ \frac{1}{2} \\\\not \ equivalent[/tex]

2/3 and 12/9 are not equivalent since 12/2 isn’t equal to 9/3.
➡️ 6 isn’t equal to 3.

20/40 and 45/55 are not equivalent since 45/20 isn’t equal to 55/40.
➡️ 2.25 isn’t equal to 1.375.


20/40 and 4/8 are equivalent since 20/4=40/8
➡️ 5=5


5/5 and 25/50 are not equivalent since 25/5 isn’t equal to 50/5.
➡️ 1 isn’t equal to 10

A businessman spends 1/5 of his travel expense funds on a hotel room and 4/10 on airfare. What percentage of his travel expenses are left over?

Answers

1/5= 20/100
4/10= 40/100
20+40= 60
100-60= 40
So he has 40% remaining

Answer: 40%

Step-by-step explanation:

1/5=20%, 4/10=40%. 20 + 40 = 60. [ 100% - 60% = 40%]

You are charged $9.33 total for a meal, assume the 7% sales tax, how much was the menu price of this item?

I have already tried
$8.68
$8.71
$8.67
all were wrong ​

Answers

Answer:

$8.71.

Step-by-step explanation:

Given that you are charged $ 9.33 total for a meal, assuming the 7% sales tax, to determine how much was the menu price of this item, the following calculation must be performed:

100 + 7 = 107

107 = 9.33

100 = X

100 x 9.33 / 107 = X

933/107 = X

8.71 = X

Therefore, the menu price of this item was $ 8.71.

Given right angle ABC, what the value of tan(A)?
5/13
12/13
12/5
13/12
need answer asap

Answers

Hi there!

[tex]\large\boxed{12/5}}[/tex]

tan (angle) = Opposite side / Adjacent side, so:

Tan (A) = opposite side / adjacent side

= 24 / 10

Simplify:

= 12 / 5

1-0.4^n>=0.99 howwwwwwwwwwwwwwwwwwwwwwwwww

Answers

Answer:

n>=6

Step-by-step explanation:

1-0.4ⁿ>=0.99

1-0.99>=0.4ⁿ

0.4ⁿ<=0.01

Apply log10:

Log10(0.4ⁿ)<=log10(0.01)

n×log10(0.4)<=log10(0.01)=-2

Because log10(0.4)=-0.39794 is negative we get:

n>=5.028.

Since n is integer, we have n>=6

Other Questions
Under a fixed exchange rate regime, the government of the country is officially responsible for ___ How do you graph this ,helppppppppp Write the equation of the line perpendicular to 7y - 35x = 21 that passes through the point (10,-8). (f) Find the present value of an investment that will pay $3,000 at the end of Years 10, 11, and 12. Use a discount rate of 8%. Find the value of x pls help A computer monitor is listed as being 22 inches. This distance is the diagonal distance across the screen. If the screen measures 12 inches in height, what is the actual width of the screen to the nearest inch?22 inches18.43 inches25.05 inches32.5 inches write a letter to your friend to join a inviting on picnic Write an application to your headteacher informing him/her about your study tour. Include all your plans in it . Also ask for the help you would like to have from the school. You can invite him/her to join your study tour , too Opponents of student cell phone use in schools claim that cell phones distract from learning. However, cell phones allow students the ability to read and download multiple texts, perform advanced calculations, create multimedia presentations, and access information. Cell phones are a great educational tool for students to use.Which of the following is most likely the main idea of this passage?Cell phones are a great form of entertainment.Cell phones should be banned from schools.Cell phones should be allowed in schools.Cell phones can do more today than they ever have before. Which scenario is an accurate example of how branches of the federalgovernment can check each other's power? Sylvain spent a total of 18 hours on English and math homework over the last 3 weeks. He spent twice as much time on English than on math. Let x represent the number of hours spent on English and y represent the number of hours spent on math.x + y = 18. x = 2 y.What is the solution of the system of equations?(12, 6)(6, 12)(9, 9)(15, 3) A boy is moving a stone in a horizontal circle by mean of a thread attached to ut. The length if the threads us 1 m and the weight of the stone is 0.02 kg . The maximum tension of the thread can withstand us 1 10 ^-4 N. The boy is gradually increasing the speed if rotation. At what speed if the stone does the thread break ? which statement describes an example of a narrative?A. a teacher gives her class instructions on how to complete their upcoming science projects.B. a child tells his friends about the exciting adventures he had over summer vacation.C. a father helps his son with his homework by explaining the answer to a complex math proble.D. a business owner gives details about his company's financial status and hiring history. What is the coefficient of x^3 in the expansion of (2x3)^5?Help please!! need help pleaseee,question is in the pic Tony is 3 years older than Marie. This can be modeled with the following equation Read the following quote and then identify the literary device that is being used.LENNOX: So that, I say,[Macbeth] has borne all things wellunderstatementsatirecomplimentsarcasm what is reductionasim Which statement describes William Blackstone's contribution to governmentin the United States?O A. He spoke out in favor of replacing the Articles of Confederationwith the U.S. Constitution.B. He introduced English translations of Enlightenment writings tothe United States.O C. He insisted that government be separated into three roughly equalbranches.O D. He helped establish many of the philosophies behind the U.S. legalsystem. Laura: Eva es muy simptica, __________?Marc: S, ella es muy simptica.A. por lo tantoB. nadaC. ningunoD. no es cierto